Kraft- und Energiebeziehung: bei zeitabhängiger Kraft

Die äquivalenten Probleme finden sich auch im Marion-Problem 7-22 und in anderen formalen Lehrbüchern der klassischen Mechanik. Hier möchte ich wissen, warum die Lehrerlösung und einige Websites diese Art von Ansatz bieten.


Ein Teilchen der Masse m bewegt sich unter dem Einfluss einer Kraft in einer Dimension

F ( X , T ) = k X 2 e X P ( T / τ )
wobei k und τ positive Konstanten sind.
Was ich tun möchte, ist seine Lagrangian zu berechnen.

F ( X , T ) = U ( X , T ) X ,

Zuerst weiß ich, dass diese Art von Ansatz (The F = U gilt nur für konservative Kräfte, ich . e , Wegunabhängig, Positionsabhängigkeitspotential), ist in der Tat gültig

Viele ähnliche Lehrbücher und ihre oben verwendete Lösung beziehen sich auf die erneute Ableitung U , und berechnen L Und H .

Ich denke, ihr Zweck ist es, das zu zeigen, selbst wenn H wird geschrieben als H = T + U , Das H unterscheidet sich von E . ich . e , wenngleich H wird geschrieben als H = T + U , da die Kraft explizit zeitabhängig ist, H ist nicht mehr E .

Ich verstehe das Verfahren, akzeptiere aber den ersten Teil nicht. Im Allgemeinen (?) Weiß ich, dass zeitabhängige Kraft nicht konservative Kraft ist. Liege ich falsch? Tut F = U X hält sowas immer?


vgl. aus Wiki

Mathematische Beschreibung[Bearbeiten] Ein überall im Raum (oder innerhalb eines einfach zusammenhängenden Raumvolumens) definiertes Kraftfeld F wird als konservative Kraft oder konservatives Vektorfeld bezeichnet, wenn es eine dieser drei äquivalenten Bedingungen erfüllt:

  1. Die Locke von F ist der Nullvektor:

    × F = 0 .

  2. Es gibt kein Netzwerk (W), das von der Kraft geleistet wird, wenn ein Teilchen durch eine Flugbahn bewegt wird, die an derselben Stelle beginnt und endet:

    W C F D R = 0.

  3. Die Kraft kann als negativer Gradient eines Potentials geschrieben werden,

    Φ : F = Φ .

Antworten (1)

Wenn Sie sich fragen, ob F = U X gilt, können Sie die entsprechende Bedingung überprüfen, die Sie gepostet haben, × F = 0 . In diesem Fall ist die Kräuselung Null, sodass Sie die Kraft als negativen Gradienten eines Potentials schreiben können. Dies scheint zu funktionieren, da der del-Operator keine Zeit berücksichtigt, sondern nur den Pfad. Hier kommt dann der Hamiltonian ins Spiel.

In diesem Fall sagen sie das nicht H E Wegen der Zeitabhängigkeit. Seit H = U + T Und E = U + T , Hier H = E (obwohl dies nicht immer der Fall ist). Da H explizit Zeit enthält, ist H keine Erhaltungsgröße. Seit H = E , Energie ist daher nicht erhalten, obwohl die Kraft konservativ zu sein schien. Das demonstrierten sie. Wenn H E , dann wäre H, das keine Erhaltungsgröße ist, irrelevant dafür, ob Energie erhalten wurde oder nicht.

Ich finde es schwierig, diese Kraft konservativ zu finden, was ist mit dem Netzwerk, wenn das Teilchen zum Startort hin und her geht? Das Netzwerk wird sicherlich nicht Null sein (die Kraft ändert sich mit der Zeit entlang des Pfades, sie ist eine Funktion von t, also ändert sie sich, wenn Sie zurückkommen)
Der Curl ist nur für 3D-Vektorfelder definiert. Ich halte es nicht für sinnvoll, die von Ihnen erwähnte Curl-Bedingung für diese 1D-Kraft zu verwenden.
Bruce, Sie haben Recht, diese Kraft ist nicht konservativ. Dies wird hier gezeigt, wenn sie sagen, H = E und H ist nicht erhalten, weil es Zeit enthält. Da die Wellung nur in drei Dimensionen definiert ist, können Sie die Wellung in einer Dimension annehmen, wobei zwei der Komponenten Null sind. Obwohl ich mich irren könnte, denke ich, dass das Gehen auf eine Dimension auf diese Weise und das Zeigen der Locke als Null immer noch eine konservative Kraft impliziert (obwohl in diesem Fall die Zeitabhängigkeit es nicht konservativ macht).
Ich frage mich, ob F = -nabla Phi für nichtkonservative Kraft gültig ist. In Wiki heißt es, dass diese Art von Ansatz nur für konservative Gewalt gilt. In meiner obigen Frage geht es in der Tat um die potentielle Energie-Kraft-Beziehung, die auch für zeitabhängige Kraft gilt, was umständlich erscheint
die eigentliche Definition ist (2). (1) und (3) sind nur dann äquivalent zu (2), wenn das Feld nur eine Funktion der Position ist.